A limit tends to Euler’s constant

euler-mascheroni-constantlimitssequences-and-series

Is this limit obvious that it is tended to Euler's constant?

$$\lim_{n \to \infty}\sqrt{\sum_{k=1}^{n}H_{k}\left(\frac{1}{k}+\frac{1}{k+1}\right)}-\ln n=\gamma$$

Where $H_k$ is the Harmonic number and $\gamma$; Euler's constant

Best Answer

Note that using Telescoping Series $$ \begin{align} \sum_{k=1}^nH_k\left(\frac1k+\frac1{k+1}\right) &=\sum_{k=1}^nH_k\left(H_{k+1}-H_{k-1}\right)\tag1\\ &=H_{n+1}H_n-H_1H_0\tag2\\[9pt] &=H_{n+1}H_n\tag3 \end{align} $$ Furthermore, since Harmonic Numbers are monotonically increasing, $$ H_n^2\le H_{n+1}H_n\le H_{n+1}^2\tag4 $$ and by the definition of the Euler-Mascheroni Constant, $$ \lim_{n\to\infty}(H_n-\log(n))=\lim_{n\to\infty}(H_{n+1}-\log(n))=\gamma\tag5 $$ The Squeeze Theorem completes the answer.

Related Question